This is an NA question.
This is a very difficult argument to understand substantively. Let’s first approach this question by breaking it up into its parts. Conveniently, it progresses in order from minor premise to major premise to main conclusion. The two conclusion indicator words “thus” and “therefore” help us recognize this.
There may be a way to get to the right answer without fully understanding the argument substantively, but I wouldn’t bank on it. You’ll have to pick up in the main conclusion the new reference to “other characteristics.” What other characteristics? We’ve only talked about a star’s brightness. So the “other characteristics,” whatever they may be, had better be determinable.
If you picked up on that, then you’re probably down to Answer Choice (B) and Correct Answer Choice (D). The problem with (B) is that it’s too specific and hence unnecessary. We don’t need “differences in the elements each is burning” to be detectable from Earth. We just need some difference to be detectable from Earth, like how (D) has it.
Okay, but let’s back up and try to actually understand this argument.
The minor premise tells us that the distance between Earth and a distant galaxy overwhelms the distance between Earth and any object in that distant galaxy. Imagine you’re on a tiny island and we’re trying to measure the distance between you and two people on a different faraway tiny island. They’re not equidistant from you. One of them is actually closer, because, say, one is standing at the shore and the other is standing on the other side of the island. But the first sentence is saying that the distance between the two islands is so vast that that’s the only thing that really matters. The islands themselves are so small and the vast space between the islands so large that it hardly matters where anyone is standing on their islands. That difference is so small that it’s negligible.
So it follows that if two stars are in the same distant galaxy (two people on the same distant island), then the distance between those two stars to Earth will in effect be the same (because whatever difference is negligible). But if we still observe a difference in their brightness, that difference can’t be due to their (negligibly different) distance. It must be due to their actual brightness as in how bright they’re actually burning and not just how bright they appear to be.
Now the argument reaches for its main conclusion. It concludes that we should be able to figure out the correlation between two stars' relative actual brightness and the two stars’ other characteristics. We see “other characteristics” appear out of nowhere. We did talk about brightness and how if two stars are in the same distant galaxy, we can in effect treat them as being equidistant from Earth in terms of their brightness. But in order to correlate their brightness with “other characteristics,” we first have to be able to detect and measure those “other characteristics.”
Again, this is what Correct Answer Choice (D) picks up on. It says that there are stars in distant galaxies that have characteristics, other than brightness, discernible from Earth. This must be true. If this were false, that would mean that we could discern only a distant star’s brightness and nothing else. If that’s true, then we would be unable to correlate brightness with anything else.
Note that (D) doesn’t care to specify what the “characteristics” are. That’s good because the conclusion didn’t care to specify either. This is what makes Answer Choice (B) unnecessary. While (B) would certainly strengthen the argument, we don’t need the differences in the elements each is burning to be detectable from Earth. Who knows what “other characteristics” the conclusion wanted to correlate with brightness? Maybe it’s the elements that each is burning. Maybe it’s the color of the stars, their size or mass, or their temperature. It could be any of those characteristics that need to be detectable from Earth.
Answer Choice (A) says that if two stars are in two different galaxies... Eh, we can stop. We don’t care. The argument cares about two stars in the same distant galaxy. (A) talks about two stars in two different (near or far?) galaxies. Whatever else (A) is about to say for these two stars will be irrelevant so you should move on to the next answer.
But for review, we can negate this and see that it has no impact on the argument. So what if it is possible to determine their distances from Earth? That doesn’t hurt the argument.
Answer Choice (C) is similarly irrelevant for it talks about stars in our own galaxy. Again, for the same reasons as in (A), we don’t care. We should move on.
If we negate (C), that’s just fine for the argument. It would be absolutely bizarre if all the stars in the Milky Way were all approximately the same distance from Earth but it wouldn’t affect the argument.
Answer Choice (E) can also be similarly eliminated as soon as you see that it’s talking about stars that are significantly different in distance from Earth. The argument contemplated two stars that are not significantly different in distance from Earth. That’s what the sub-argument established by placing the two stars in distant galaxies to begin with. We should move on.
(E) goes on to say that if there are significant differences in how far away two stars are from Earth, then those stars will differ significantly in apparent brightness. This isn’t required. It’s fine for the argument if two stars of significant difference in distance from Earth are about the same in brightness. The argument contemplated two stars of insignificant difference in distance but significant difference in brightness. After all, a major assumption of the argument is that distance is only one factor in determining a star’s brightness.
This is a Sufficient Assumption question.
The argument starts with a conditional: a thriving population of turtles in a pond requires beneficial conditions at the pond.
thriving → beneficial
Wallakim Pond, we’re told, has acidic water.
acidicw
We’re also told that Sosachi Pond doesn’t but the conclusion doesn’t care about Sosachi and so we shouldn’t either.
Finally, the conclusion says that the population of turtles at Wallakim Pond must not be thriving.
/thrivingw
Let’s put this all together.
thriving → beneficial
acidicw
_________________
/thrivingw
Looking at the conclusion, you can see that the argument is trying to contrapose on the conditional. It’s trying to fail the “beneficial” condition. If it’s successful in doing that, then we can conclude “/thriving.” But the problem is that the only other premise doesn’t hook up to “/beneficial.” We don’t know what “acidic” means for turtles. Is that beneficial for them or not? If we’re able to establish that “/acidic” is a necessary condition, then this argument becomes valid:
thriving → beneficial → /acidic
acidicw
_________________
/thrivingw
This is what Correct Answer Choice (E) gives us. It says that the conditions of a pond are beneficial only if the water is not acidic. That’s exactly what we’re looking for: beneficial → /acidic.
Answer Choice (A) says that if the water is not acidic, then the conditions are beneficial. /acidic → beneficial. That’s the sufficiency-necessity confused version of (E).
Answer Choice (B) says that the acidity of water is the most important factor that determines whether the population of turtles will be thriving. But that doesn’t tell us whether acid is good or bad. It just says it’s powerful. In which direction? Even if (B) said that it’s in the bad direction, it would merely strengthen the argument, which would still fall short of the SA requirement.
This is what we in effect get in Answer Choice (C). It says that the conditions at Sosachi are more beneficial than the conditions at Wallakim. We have to assume that all other conditions are held equal between Sosachi and Wallakim. On that assumption, we can infer that the difference is caused by the difference in their waters’ acidity. But even then, it just means that acidity is relatively less beneficial.
Answer Choice (D) says that Wallakim would have a thriving population if the water were not acidic. That translates to /acidic → thriving. But that doesn’t fit what we’re looking for.
That means it’s the acidity in the water that’s causing the population to not be thriving.
This is a PSA question.
The argument begins with the conclusion that some of the rare pygmy bears should be moved from their native island to the neighboring island. Naturally, we wonder why. The rest of the argument supplies the premises. First, we learned that they are at risk of extinction owing to habitat loss. Second, we learned that the neighboring island is the only place that has a similar habitat. Hence, moving them is the only viable chance of saving them from extinction. That's a sub-conclusion/major premise. The main conclusion is the first sentence. We should move them.
This PSA question is just like most other PSA questions. The argument presents a P and arrives at a C. Our job is to find in the answers a P → C rule or bridge.
We can say something like if an action is the only viable method of saving an endangered species, then we should take that action. Keep in mind that PSA answers can be stated very specifically or very generally. Overinclusiveness is not a problem for this question type.
Correct Answer Choice (C) gets the job done. It says if a species is in danger of extinction, whatever is most likely to prevent the extinction should be undertaken. The premises trigger the sufficient condition because the rare pygmy bears are explicitly said to be at risk of extinction. The conclusion satisfies the necessary condition. Moving them to the neighboring island is the only viable chance and therefore it is the most likely way to prevent extinction. Therefore, it should be undertaken.
Answer Choice (B) can be eliminated on the basis of its logic alone, as is commonly the case for wrong answers on PSA questions. It says rare animals should not be moved from one habitat to another unless these habitats are similar to one another. This stipulates a necessary condition on the movement, not a sufficient condition on the movement. That's a problem for us because the conclusion wants to move these animals. Do the Group 3 translation on the logical indicator “unless.” If the habitats are not similar to one another, then the animals should not be moved. Satisfying the sufficient condition here only allows us to draw the conclusion that these animals should not be moved.
Answer Choice (E) can be eliminated because it's too weak. It's better than (B) in the sense that there is no logical issue. It says if an animal's original habitat is in danger of being lost, then it is permissible to try to find a new habitat for the animal. That's fine, the premises satisfy the sufficient condition, which allows us to draw the conclusion that it is permissible to try to find a new habitat for the pygmy bears. But that doesn't mean we should do it. Permissible doesn't imply should. This is too weak.
Answer Choice (A) says some species are more deserving of protection than other species. This is a truism. Which species are more deserving of protection than others? We don't know. Even if we did, what manner should the protection take? Again, we don't know.
Answer Choice (D) says the rarer a species of plant or animal is, the more that should be done to protect that species. This allows us to draw conclusions about preservation priorities. If we know that the rare pygmy bear is rarer than, say, the panda bear, then according to (D), we should afford priority and do more to protect the pygmy bears. But how is this relevant to the argument? We’re not concerned about whether we're doing too much or too little for the pygmy bears in comparison to some other endangered species.
This is a PSA question.
The argument begins with the conclusion that some of the rare pygmy bears should be moved from their native island to the neighboring island. Naturally, we wonder why. The rest of the argument supplies the premises. First, we learned that they are at risk of extinction owing to habitat loss. Second, we learned that the neighboring island is the only place that has a similar habitat. Hence, moving them is the only viable chance of saving them from extinction. That's a sub-conclusion/major premise. The main conclusion is the first sentence. We should move them.
This PSA question is just like most other PSA questions. The argument presents a P and arrives at a C. Our job is to find in the answers a P → C rule or bridge.
We can say something like if an action is the only viable method of saving an endangered species, then we should take that action. Keep in mind that PSA answers can be stated very specifically or very generally. Overinclusiveness is not a problem for this question type.
Correct Answer Choice (C) gets the job done. It says if a species is in danger of extinction, whatever is most likely to prevent the extinction should be undertaken. The premises trigger the sufficient condition because the rare pygmy bears are explicitly said to be at risk of extinction. The conclusion satisfies the necessary condition. Moving them to the neighboring island is the only viable chance and therefore it is the most likely way to prevent extinction. Therefore, it should be undertaken.
Answer Choice (B) can be eliminated on the basis of its logic alone, as is commonly the case for wrong answers on PSA questions. It says rare animals should not be moved from one habitat to another unless these habitats are similar to one another. This stipulates a necessary condition on the movement, not a sufficient condition on the movement. That's a problem for us because the conclusion wants to move these animals. Do the Group 3 translation on the logical indicator “unless.” If the habitats are not similar to one another, then the animals should not be moved. Satisfying the sufficient condition here only allows us to draw the conclusion that these animals should not be moved.
Answer Choice (E) can be eliminated because it's too weak. It's better than (B) in the sense that there is no logical issue. It says if an animal's original habitat is in danger of being lost, then it is permissible to try to find a new habitat for the animal. That's fine, the premises satisfy the sufficient condition, which allows us to draw the conclusion that it is permissible to try to find a new habitat for the pygmy bears. But that doesn't mean we should do it. Permissible doesn't imply should. This is too weak.
Answer Choice (A) says some species are more deserving of protection than other species. This is a truism. Which species are more deserving of protection than others? We don't know. Even if we did, what manner should the protection take? Again, we don't know.
Answer Choice (D) says the rarer a species of plant or animal is, the more that should be done to protect that species. This allows us to draw conclusions about preservation priorities. If we know that the rare pygmy bear is rarer than, say, the panda bear, then according to (D), we should afford priority and do more to protect the pygmy bears. But how is this relevant to the argument? We’re not concerned about whether we're doing too much or too little for the pygmy bears in comparison to some other endangered species.
The question stem says of the following judgments, which one most closely conforms to the principle above? This is a rarer type of question though we have seen it plenty before. They're asking us to take the principle in the stimulus which is a conditional statement and push it into the arguments in the answers to see where it fits. But that’s like a PSA question. Instead of the stimulus containing an argument searching for a conditional in the answer, it's the other way around. The stimulus contains a conditional searching for an argument. This is a cosmetic difference.
The stimulus lays down two jointly sufficient conditions for justified governmental interference with an individual’s actions. The two conditions are:
- The action would increase the likelihood of harm to others; and
- The action is not motivated by a desire to help others.
If both conditions are met, then the government is justified in interfering with the individual’s action.
As an aside, note that these two conditions each cover a different kind of consideration. The first looks at the consequences of the action. Will this action harm others? The second looks at the intent of the action. What motivated the action? In general, considerations of morality tend to fall into these two buckets of consequences and intent.
Back to the task at hand. Given that this is a PSA question, it’s important to note which conclusions are reachable and which are unreachable.
A reachable conclusion is that the government is justified in interfering with the individual’s action. To reach this conclusion, we just have to show that (1) and (2) are satisfied.
An unreachable conclusion is that the government is not justified in interfering with the individual’s action. There’s simply nothing we can show to trigger the conditional in that way. If we wanted to reach the unjustified conclusion, we need to know what the necessary conditions of justified are, fail those conditions, then contrapose back.
This analysis is helpful in eliminating Answer Choice (A). It tries to conclude that the government is unjustified in interfering with Jerry’s moviemaking. We don’t need to read the rest of the argument. There’s nothing that the premise can state that will make use of the conditional to reach this conclusion. (A) is therefore wrong on its logic alone. In other words, it makes a sufficiency-necessity confusion, the oldest mistake in the book. We can stop here, but for review, look at the premise. It says that Jerry’s action (moviemaking) doesn’t harm and won’t increase the likelihood of harming anyone. It also says that it is motivated by a desire to help others. So it fails both (1) and (2). But failing sufficient conditions just makes the rule go away. It doesn’t trigger anything. Yet, (A) thinks it triggers the failure of the necessary condition. That’s textbook sufficiency-necessity confusion.
Contrast this with Correct Answer Choice (E). It concludes that the government is justified in preventing Jill from giving her speech. That’s a reachable conclusion. We just need to show that Jill’s speech satisfies (1) and (2). And it does. Her speech “would most likely have caused a riot and people would have gotten hurt.” That’s physical harm to others. And her speech was “to further her own political ambitions.” That’s a selfish motivation and hence not a motivation to help others.
Answer Choice (B) concludes that the government is justified in fining the neighbor for not mowing his lawn. That’s a reachable conclusion. We just need to show that the neighbor’s not mowing his lawn satisfies (1) and (2). (1) is problematic. It’s not pleasant to look at an unkempt lawn, but that’s not physical harm to others. We don’t need to consider (2) but probably the neighbor’s decision to not mow his lawn was selfishly motivated. He was probably just feeling lazy.
Answer Choice (C) concludes that the government is justified in requiring motorcyclists to wear helmets. That’s a reachable conclusion. We just need to show that motorcyclists’ not wearing helmets satisfies (1) and (2). Again, we have a problem for (1). It’s not clear that their failure to wear helmets would increase harm to others, whatever the consequences of harm are for themselves. We don’t need to consider (2) but probably their decision to not to wear helmets was selfishly motivated. They probably were feeling lazy, wanted to look cool, or have a death wish.
Answer Choice (D) concludes that the government is justified in suspending Z’s license to test new drugs. That’s a reachable conclusion. We just need to show that Z’s testing new drugs satisfies (1) and (2). Again, we have a problem for (1). It’s not clear that their testing of new drugs would increase harm to others. In fact, if they’re a drug company, then it’s more likely that their testing of new drugs would do just the opposite. It would help others alleviate pain and suffering. We don’t need to consider (2) but here the argument makes explicit that their motivation is selfish and not to help others.
This is a Most Strongly Supported question.
The stimulus provides two different types of information. First, we’re given a correlation, which turns out to be useless. Second, we’re given a logical chain, which is what produces the inference.
The correlation is that darker honey tends to be higher in antioxidants than lighter honey.
The next piece of information, even though it's still in the same sentence, expresses a different relationship. It says that all of the most healthful strains of honey are unusually high in antioxidants. The keyword is “all” which the test writers conveniently hid in the middle of the sentence. If you catch that, you can translate this into an all statement using the conditional arrow. The set of “the most healthful strains of honey” is completely subsumed under the set of “honey that's unusually high in antioxidants”:
most healthful → unusually high in antioxidants
Finally, we learned that there are some strains of honey that come from sage nectar and are among the lightest in color, yet are also among the most healthful. This is a “some relationship,” an overlap in two sets. One of the sets is what we've already talked about: the set of the most healthful strains of honey. The other is the set of honey that comes from sage nectar and is lightest in color.
sage and among lightest ←s→ most healthful
We can chain together this “some statement” and the previous “all statement”:
sage and among lightest ←s→ most healthful → unusually high in antioxidants
This is a commonly repeating valid argument form A ←s→ B → C which produces the valid inference A ←s→ C. Translated back into English, some of the lightest strains of honey produced by bees harvesting sage nectar are unusually high in antioxidants. This is what Correct Answer Choice (A) says. Almost. (A) drops “lightest” but that’s fine. If it’s true that some of the lightest sage honey is X, then it’s also true that some sage honey is X.
Answer Choice (B) says most plants produce nectar that results in light-colored honey. This is unsupported. The information in the stimulus is consistent with most plants producing dark-colored honey or light-colored honey.
Answer Choice (C) says light-colored honey tends to be more healthful than dark honey. This is not supported (or actually, a bit anti-supported). All we know is that darker honey tends to be high in antioxidants. We also know that the most healthful honeys are all unusually high in antioxidants. This weakly suggests that it's the antioxidants that are causally responsible for the healthful effects. If we take that to be true, then (C) is actually anti-supported. But we don’t have to because this is just an MSS question and being unsupported is good enough to eliminate this answer.
Answer Choice (D) says certain strains of honey produced by bees harvesting primarily sage nectar are unusually low in antioxidants. This is unsupported. It’s a tempting answer because we know that sage nectar produces “among the lightest strains of honey” and we also know that there is a general correlation between honey being light and it having less antioxidants. But we also have enough information to infer that sage is an exception to the correlation, because we know that sage-produced light honey is among the most healthful strains of honey and we further know that the entire set of the most healthful strains of honey is subsumed under the set of honey that is unusually high in antioxidants.
Answer Choice (E) says the strain of honey that has the highest antioxidant content is a light-colored honey. This is unsupported. It could be true but it also could be false. We only have information in the stimulus about the set of honey that is among the lightest or among the most healthful or is unusually high in antioxidants. We have no information about the specific strains of honey at any of the extremes of those spectrums.
A
Some strains of honey produced by bees harvesting sage nectar are unusually high in antioxidants.
B
Most plants produce nectar that, when harvested by bees, results in light-colored honey.
C
Light-colored honey tends to be more healthful than dark honey.
D
Certain strains of honey produced by bees harvesting primarily sage nectar are unusually low in antioxidants.
E
The strain of honey that has the highest antioxidant content is a light-colored honey.
Further Explanation
This is a Most Strongly Supported question.
The stimulus provides two different types of information. First, we’re given a correlation, which turns out to be useless. Second, we’re given a logical chain, which is what produces the inference.
The correlation is that darker honey tends to be higher in antioxidants than lighter honey.
The next piece of information, even though it's still in the same sentence, expresses a different relationship. It says that all of the most healthful strains of honey are unusually high in antioxidants. The keyword is “all” which the test writers conveniently hid in the middle of the sentence. If you catch that, you can translate this into an all statement using the conditional arrow. The set of “the most healthful strains of honey” is completely subsumed under the set of “honey that's unusually high in antioxidants”:
most healthful → unusually high in antioxidants
Finally, we learned that there are some strains of honey that come from sage nectar and are among the lightest in color, yet are also among the most healthful. This is a “some relationship,” an overlap in two sets. One of the sets is what we've already talked about: the set of the most healthful strains of honey. The other is the set of honey that comes from sage nectar and is lightest in color.
sage and among lightest ←s→ most healthful
We can chain together this “some statement” and the previous “all statement”:
sage and among lightest ←s→ most healthful → unusually high in antioxidants
This is a commonly repeating valid argument form A ←s→ B → C which produces the valid inference A ←s→ C. Translated back into English, some of the lightest strains of honey produced by bees harvesting sage nectar are unusually high in antioxidants. This is what Correct Answer Choice (A) says. Almost. (A) drops “lightest” but that’s fine. If it’s true that some of the lightest sage honey is X, then it’s also true that some sage honey is X.
Answer Choice (B) says most plants produce nectar that results in light-colored honey. This is unsupported. The information in the stimulus is consistent with most plants producing dark-colored honey or light-colored honey.
Answer Choice (C) says light-colored honey tends to be more healthful than dark honey. This is not supported (or actually, a bit anti-supported). All we know is that darker honey tends to be high in antioxidants. We also know that the most healthful honeys are all unusually high in antioxidants. This weakly suggests that it's the antioxidants that are causally responsible for the healthful effects. If we take that to be true, then (C) is actually anti-supported. But we don’t have to because this is just an MSS question and being unsupported is good enough to eliminate this answer.
Answer Choice (D) says certain strains of honey produced by bees harvesting primarily sage nectar are unusually low in antioxidants. This is unsupported. It’s a tempting answer because we know that sage nectar produces “among the lightest strains of honey” and we also know that there is a general correlation between honey being light and it having less antioxidants. But we also have enough information to infer that sage is an exception to the correlation, because we know that sage-produced light honey is among the most healthful strains of honey and we further know that the entire set of the most healthful strains of honey is subsumed under the set of honey that is unusually high in antioxidants.
Answer Choice (E) says the strain of honey that has the highest antioxidant content is a light-colored honey. This is unsupported. It could be true but it also could be false. We only have information in the stimulus about the set of honey that is among the lightest or among the most healthful or is unusually high in antioxidants. We have no information about the specific strains of honey at any of the extremes of those spectrums.
This is an Inference question.
The question stem says “properly inferred” from the sociologist's perspective. Inference from others' perspective is a question type that we see more often in RC.
The stimulus starts by telling us what rational choice theory says about what causes support for political parties. It says that popular support for political parties is caused by individual voters making deliberate decisions to support those parties whose policies they believe will economically benefit them. In other words, individuals' beliefs about the economic consequences of a particular party's policies cause those individuals to support those parties. This causal relationship is what is meant by “sufficiently explained.”
But the sociologists don't agree. They oppose rational choice theory on the premise that a complex phenomenon like the rise of a political organization or party cannot be caused by a simple phenomenon.
What is this “simple phenomenon”? It must be the individual voters making economic decisions to support political parties, which implies that it must not be a complex phenomenon. This is what Correct Answer Choice (A) says. Sociologists believe that economically motivated decisions by voters need not constitute a complex phenomenon. We are getting hints of an NA question. Note how (A) could have stated this much more strongly. Economically motivated decisions by voters constitute a simple phenomenon. That would have been correct as well. But the test writers took it one step further and stated an inference of that statement.
Answer Choice (B) says a complex phenomenon generally will have many complex causes. This is unsupported. The sociologists only said that a complex phenomenon cannot be caused by a simple phenomenon. This leaves open several possibilities. Perhaps they believe that a complex phenomenon can be caused by many simple phenomena. Or perhaps they believe that a complex phenomenon can be caused by a single complex phenomenon. We’d have to dismiss those alternatives without warrant in order to arrive at (B).
Answer Choice (C) says political phenomena often have religious and cultural causes as well as economic ones. This is even more unsupported. Note the same reasoning in (B) applies here. Additionally, (C) draws an inference to religious and cultural causes on the basis of nothing.
Answer Choice (D) says popular support for political parties is never a complex phenomenon. This is anti-supported. The sociologist called the rise of a political organization a complex phenomenon. Within the context of the stimulus, the rise of the political organization is synonymous with popular support for a political party.
Answer Choice (E) says the decisions of individual voters are not usually influenced by their beliefs about which policies will yield them the greatest economic advantage. This is unsupported. The stimulus talks about a narrow political relationship. It examines the causes of the rise of popular political parties. (E) talks about a much broader political relationship, the causes of individual voting decisions. The stimulus has very little to say about what generally causes (influences) or doesn't cause voters to cast their vote one way or another.
This is an Inference question.
The question stem says “properly inferred.” This is a challenging question because there is so much information in the stimulus that is all connected. That can easily induce panic as you scramble to draw all the connections and valid inferences. Strategically, you shouldn't do that. The more connected information a stimulus contains, the more valid inferences there are to be drawn, the less you are able to anticipate the correct answer choice. In stimuli like those, POE is the better approach.
The first line in the stimulus about the study of primates being interesting is the only irrelevant fact. Everything else is fair game.
Only primates have opposable thumbs. Lemurs are lower primates (a subset of primates.) And lemurs are the only primates indigenous to Madagascar. Some species of lemurs are the only living lower primates that are diurnal. They go ahead and define diurnal for us but the answer choices never swapped out the term for its definition so we don't need to pay attention to it. Finally, all higher primates (a subset of primates) are thought to have evolved from a single diurnal species of lower primates.
Lots of information. Let’s POE.
The last piece of information is what sets up the trap in Answer Choice (A). It says that the chimpanzee, a higher primate, evolved from the lemur. This is not a proper inference. This is unsupported. All we can say is that the chimpanzee, being a higher primate, is thought to have evolved from a single diurnal species of lower primates. Which one, though? Must it be the lemur because the lemur is a diurnal species of lower primates and it is the only living one? No. Being alive isn’t required. We’re talking about evolution here. Most ancestor species are extinct. There may well have been other extinct, diurnal species of lower primates. One of those extinct species may well be the evolutionary starting point of all higher primates.
Correct Answer Choice (B) says no primates indigenous to Madagascar are diurnal higher primates. We can transform this into the following logically equivalent claim: all primates indigenous to Madagascar are not diurnal higher primates. This must be true. The stimulus says that the only primates indigenous to Madagascar are lemurs and that lemurs are all lower primates. It is implied that lower primates cannot be higher primates, diurnal or otherwise. This is the conditional chain: prim-indig-M → lemur → low-prim
Answer Choice (C) says no higher primate is nocturnal. This is unsupported. We simply have no idea if higher primates are nocturnal or diurnal or anything else. The only piece of information we have about higher primates is that they are thought to have evolved from a single diurnal species of lower primates.
Answer Choice (D) says there are some lemurs without opposable thumbs. This is unsupported. The stimulus says only primates have opposable thumbs. That means if something is not a primate then it doesn't have opposable thumbs. But lemurs are primates. Sufficient condition failed, rule goes away.
Answer Choice (E) says there are no nocturnal lemurs. This is unsupported. The stimulus tells us that some species of lemurs are diurnal. Maybe all species of lemurs are diurnal, maybe not.
This is a Weaken question.
The author concludes that the government should require drug companies to notify consumers of all known drug-related interactions. Why? Because even relatively minor drug-related interactions can still be harmful to patients. That’s the major premise. Why should we believe this? The author provides an example: aspirin and fruit juice interact to render the aspirin ineffective. People who are unaware of this end up taking an incorrect dosage.
The logic of this argument is cost-benefit analysis. We consider a cost and from that consideration draw a conclusion about what should be done. As with all cost-benefit analysis questions, there are other factors to consider.
Correct Answer Choice (A) provides a counterbalancing consideration. It says that providing information on minor drug-related interactions would detract from a patient’s attention to serious interactions. That seems like a pretty compelling cost which weighs against the stated benefit in the stimulus. In fact, it seems like it outweighs the benefit, though that isn’t required in order to weaken the reasoning.
Answer Choice (B) says many drugs have fewer documented drug-related interactions than does aspirin. This is irrelevant. The argument used aspirin merely as an example. It never assumed that aspirin was the drug with the least documented interactions.
Answer Choice (C) says providing information about all drug-related interactions would result in only negligible price increases for consumers. This is a consideration on the benefit side of the scales. This wouldn’t weaken the argument.
Answer Choice (D) says current research is such that many drug-related interactions have not yet been identified. Of course not. Why should we expect we’d have completely identified all the interactions? But the mere fact that there are interactions that we don’t yet know about weighs on neither side of the scales. If the concern here is that there may be some as yet unidentified interaction X that’s really dangerous, what can we do? We certainly can’t print X on the label because we don’t know about X. That’s what it means to be unidentified.
Answer Choice (E) says pharmacists usually draw patients’ attention to printed warnings that are provided with drugs. This also wouldn’t sway the argument in either direction. Pharmacists are merely acting as a highlighter, reinforcing the already printed warnings. We’re talking about whether to add additional warnings onto the label.